Funzionale a caso

Polinomi, disuguaglianze, numeri complessi, ...
Rispondi
FloatingPoint
Messaggi: 22
Iscritto il: 18 lug 2016, 20:34

Funzionale a caso

Messaggio da FloatingPoint »

Own.
Trovare tutte le funzioni [math] tali che:
  • [math]
  • [math]
P. S. È il primo problema che propongo qui, siate clementi :wink:
Ultima modifica di FloatingPoint il 31 mag 2017, 20:48, modificato 1 volta in totale.
Avatar utente
Sirio
Messaggi: 317
Iscritto il: 08 set 2016, 22:01

Re: Funzionale a caso

Messaggio da Sirio »

Osservazioni a caso sulla tua funzionale a caso
Testo nascosto:
Ponendo $x=y$ nella prima ipotesi otteniamo:\[f\left(x^2\right)-f^2\left(x\right)=0\;\;\forall x\in\mathbb R\]Da cui:\[f\left(x^2\right)=f^2\left(x\right)\;\;\forall x\in\mathbb R\]Da cui:\[f\left(x\right)=f^2\left(\sqrt x\right)=f^2\left(-\sqrt x\right)\ge0\;\;\forall x\in\mathbb R_0^+\]Da cui:\[f(-x)=\pm f\left(x\right)\;\;\forall x\in\mathbb R^+\;\;\wedge\;\;f\left(0\right)=0\]Per la seconda ipotesi, $f$ non può essere costantemente $0$ e almeno una volta di quelle in cui è diverso da $0$ nella formula qui sopra si sceglie il segno meno.
Applicando $f\left(x^2\right)=f^2\left(x\right)$ alla prima ipotesi otteniamo che essa è equivalente a:\[\left(f\left(x\right)+f\left(y\right)\right)\left(f\left(x\right)-f\left(y\right)\right)=f\left(x+y\right)\left(f\left(x\right)-f\left(y\right)\right)\;\;\forall x;y\in\mathbb R\]Ovviamente sarebbe bello scoprire che per ogni $x\neq y$ si abbia $f\left(x\right)\neq f\left(y\right)$, perché allora avremmo la seguente cosa bella:\[f\left(x\right)+f\left(y\right)=f\left(x+y\right)\;\;\forall x;y\in\mathbb R;\;x\neq y\]Se poi si scoprisse anche che $f$ è continua potrei togliere la condizione $x\neq y$ e concludere che le funzioni che mi chiedi sono solo le rette passanti per l'origine con coefficiente angolare positivo. In particolare, dovendo valere $f\left(x^2\right)=f^2\left(x\right)$ solo l'identità. Ma... Saran vere le mie due congetture? Non mi sento in grado di rispondere, fate voi.
$T=\sqrt{\dfrac l g 12\pi}$
alex00
Messaggi: 8
Iscritto il: 18 giu 2016, 00:57

Re: Funzionale a caso

Messaggio da alex00 »

Tento una soluzione al problema, non sono sicurissimo però.
Testo nascosto:
Poniamo \(x=y=0\), otteniamo \(f(0)-f(0)^2=0 \Rightarrow f(0)=0\) o \(f(0)=1\).
Adesso poniamo solo \(x=0\), otteniamo \(f(0)=f(y)f(0)\) Sostituiamo \(f(0)=1\) e otteniamo che \(f(y)=1\) ma se la funzione fosse costante di valore 1 non soddisferebbe la seconda condizione della traccia. Dunque provo con \(f(0)=0\) che funziona. Quindi sappiamo che \(f(0)=0\).

Ora poniamo solo \(y=0\), e sapendo che \(f(0)=0\) otteniamo \(f(x^2)=f(x)^2\), dunque adesso possiamo riscrivere la traccia come:
\begin{equation}
f(x)^2-f(y)^2=f(x+y)(f(x)-f(y))
\end{equation}

Scomponendo l'LHS possiamo dividere due casi.
1) \(f(x)-f(y)=0 \Rightarrow f(x)=f(y) \Rightarrow f(x)=c\) con c costante, che però non soddisfa la condizione trovata che \(f(x^2)=f(x)^2\)
2) \(f(x)-f(y)\not=0\) e quindi dividendo ottengo \(f(x)+f(y)=f(x+y)\)

Escludendo il caso 1) ottengo la nuova condizione dal caso 2)

Dimostriamo che \(f(1)=1\), sostituiamo \(x=1\) nella condizione \(f(x^2)=f(x)^2\) e otteniamo anche qui due casi, \(f(1)=1\) e \(f(1)=0\). Possiamo escludere \(f(1)=0\) in questa maniera:

prendendo la condizione \(f(x)+f(y)=f(x+y)\) e sostituendo \(x=y=1\) otteniamo che \(f(2)=0\), così con \(x=1, y=2\) otteniamo che \(f(3)=0\) e quindi andando avanti così dovrei avere che \(f(x)=0\), questo però non soddisfa la seconda condizione della traccia.

Dunque \(f(1)=1\), quindi prendendo un numero intero n ho che \(f(n)=nf(1)\) (applicando tante volte la condizione \(f(x)+f(y)=f(x+y)\)) quindi \(f(n)=n\), questo vuol dire che se vale per un n varrà per ogni x, quindi una funzione buona è \(f(x)=x\), inoltre è l'unica perchè è l'unica che soddisfa le nostre condizioni.
matpro98
Messaggi: 479
Iscritto il: 22 feb 2014, 18:42

Re: Funzionale a caso

Messaggio da matpro98 »

Non credo tu possa passare dagli interi ai reali
alex00
Messaggi: 8
Iscritto il: 18 giu 2016, 00:57

Re: Funzionale a caso

Messaggio da alex00 »

Eh, è lì il mio dubbio.
bern-1-16-4-13
Messaggi: 78
Iscritto il: 23 mag 2015, 18:27

Re: Funzionale a caso

Messaggio da bern-1-16-4-13 »

Ciao Floatingpoint, benvenuto!! Immagino tu debba essere alle prime armi :D

Mettiamo un po' di ordine in questo topic via, ti metto qua lo sketch della soluzione
Testo nascosto:
Sia $TROLL\left(x,y\right)$
Sostituiamo $TROLL\left(x,x\right)$ per ottenere $f\left(x^2\right)=f\left(x\right)^2$, quindi andando a sostituire nel testo otteniamo $$f\left(x\right)^2-f\left(y\right)^2=f\left(x+y\right)\left(f\left(x\right)-f\left(y\right)\right)$$

Sia adesso $g^n\left(x\right)$ la reiterata $n$-esima della generica funzione $g\left(x\right)$ (con opportune generalizzazioni nel caso in cui $n\not\in\mathbb{N}$ e/o $n\not\in\mathbb{Z}$
Sia $\phi$ la funzione di Eulero
Sia $C_n$ l'$n$-esimo numero di Catalan (con opportune generalizzazioni se $n\not\in\mathbb{Z}$
Sia $\mathbb{A}$ l'insieme dei numeri algebrici.
Sia infine $\left[P\right]$ la funzione verità (che quindi vale $0$ se $P$ è falsa, $1$ se $P$ è vera).

E ora il passaggio chiave. Poniamo $$x=log^{C_{\gamma}}_{1+\phi\left(f^{f^{\gamma-1}\left(\pi\right)}\left(\left\vert\gamma\right\vert^{\gamma}\right)\right)}\left(\sum_{i=2}^{C_{\gamma}}\left[\gamma^{\gamma\log_i\gamma}\in\mathbb{A}\right]\left\vert\gamma\right\vert!\right)$$
Poniamo infine $y=x^{-1}$ mettiamo tutto in $TROLL\left(x,y\right)$ e semplificando il semplificabile si ottiene l'unica soluzione $f\left(x\right)=x\ \ \forall\ x\in\mathbb{R}$.
Ultima modifica di bern-1-16-4-13 il 02 giu 2017, 16:29, modificato 2 volte in totale.
Avatar utente
Sirio
Messaggi: 317
Iscritto il: 08 set 2016, 22:01

Re: Funzionale a caso

Messaggio da Sirio »

Tutto chiaro! Grazie mille Bern :D
$T=\sqrt{\dfrac l g 12\pi}$
FloatingPoint
Messaggi: 22
Iscritto il: 18 lug 2016, 20:34

Re: Funzionale a caso

Messaggio da FloatingPoint »

bern-1-16-4-13 ha scritto: 02 giu 2017, 11:19 Ciao Floatingpoint, benvenuto!! Immagino tu debba essere alle prime armi :D

Mettiamo un po' di ordine in questo topic via, ti metto qua lo sketch della soluzione
Testo nascosto:
Sia $TROLL\left(x,y\right)$
Sostituiamo $TROLL\left(x,x\right)$ per ottenere $f\left(x^2\right)=f\left(x\right)^2$, quindi andando a sostituire nel testo otteniamo $$f\left(x\right)^2-f\left(y\right)^2=f\left(x+y\right)\left(f\left(x\right)-f\left(y\right)\right)$$

Sia adesso $g^n\left(x\right)$ la reiterata $n$-esima della generica funzione $g\left(x\right)$ (con opportune generalizzazioni nel caso in cui $n\not\in\mathbb{N}$ e/o $n\not\in\mathbb{Z}$
Sia $\phi$ la funzione di Eulero
Sia $C_n$ l'$n$-esimo numero di Catalan (con opportune generalizzazioni se $n\not\in\mathbb{Z}$
Sia $\mathbb{A}$ l'insieme dei numeri algebrici.
Sia infine $\left[P\right]$ la funzione verità (che quindi vale $0$ se $P$ è falsa, $1$ se $P$ è vera).

E ora il passaggio chiave. Poniamo $$x=log^{C_{\gamma}}_{1+\phi\left(f^{f^{\gamma-1}\left(\pi\right)}\left(\gamma^{\gamma}\right)\right)}\left(\sum_{i=2}^{C_{\gamma}}\left[\gamma^{\gamma\log_i\gamma}\in\mathbb{A}\right]\gamma!\right)$$
Poniamo infine $y=x^{-1}$ mettiamo tutto in $TROLL\left(x,y\right)$ e semplificando il semplificabile si ottiene l'unica soluzione $f\left(x\right)=x\ \ \forall\ x\in\mathbb{R}$.
La soluzione funziona benissimo, ed effettivamente si semplifica tutto... Peccato però che f non sia definita sui complessi. Forse dovresti lavorare con le continuazioni analitiche meromorfe? Che succede poi se sono biperiodiche?
bern-1-16-4-13
Messaggi: 78
Iscritto il: 23 mag 2015, 18:27

Re: Funzionale a caso

Messaggio da bern-1-16-4-13 »

ah vabbe', ma quello è un typo, all'interno della sommatoria è ovvio che c'è un $\left\vert\gamma\right\vert!$
Modifico subito (anche se in effetti probabilmente si aggiusta anche nel modo che dici tu eh).
bern-1-16-4-13
Messaggi: 78
Iscritto il: 23 mag 2015, 18:27

Re: Funzionale a caso

Messaggio da bern-1-16-4-13 »

Ok, dopo numerosissime richieste, questa è una soluzione vera (o almeno dovrebbe) :lol:
Testo nascosto:
Sia $QUESTONONE'UNTROLL\left(x,y\right)$ l'uguaglianza del testo.
Step 0.
Ponendo $QUESTONONE'UNTROLL\left(0,y\right)$ si ottiene $f\left(0\right)=f\left(0\right)f\left(y\right)$, da cui $f\left(0\right)=0$ altrimenti $f\left(y\right)=1\ \ \forall y$ il che contraddirebbe la seconda ipotesi.
Ponendo $QUESTONONE'UNTROLL\left(x,x\right)$ si ottiene che $f\left(x^2\right)=f\left(x\right)^2\ \ \left(*\right)$, e da questa, notando che il LHS rimane invariato sostituendo $x$ con $-x$, si ottiene anche che $f\left(x\right)^2=f\left(-x\right)^2$, da cui $f\left(-x\right)=\pm f\left(x\right)$. Un altro corollario di $\left(*\right)$ è che $f\left(x\right)\ge 0\ \ \forall\ x\ge 0$.
Sostituendo nel testo la $\left(*\right)$ si ha che $$\left(f\left(x\right)-f\left(y\right)\right)\left(f\left(x\right)+f\left(y\right)\right)=f\left(x+y\right)\left(f\left(x\right)-f\left(y\right)\right)$$da cui $f\left(x\right)+f\left(y\right)=f\left(x+y\right)\ \ \forall\ x,y:\ f\left(x\right)\ne f\left(y\right)\ \ \left(**\right)$.

Step 1.
Dimostriamo che se $x\ne 0$ allora $f\left(x\right)\ne 0$ (quindi il secondo corollario di $\left(*\right)$ diventerebbe $\left(*\right)$ è che $f\left(x\right)>0\ \ \forall\ x>0$). Se per assurdo esistesse $t\ne 0:\ \ f\left(t\right)=0$ allora per il corollario della $\left(*\right)$ si ha che $f\left(\left\vert t\right\vert\right)=0$, ma allora scegliamo $\gamma$ tale che $f\left(\gamma\right)<0$ e un $k$ intero positivo tale che $k\left\vert t\right\vert>-\gamma$. Da una reiterazione della $\left(**\right)$ si ha che $f\left(\gamma\right)+kf\left(\left\vert t\right\vert\right)=f\left(\gamma+k\left\vert t\right\vert\right)$ da cui $f\left(\gamma+k\left\vert t\right\vert\right)=f\left(\gamma\right)<0$, assurdo perché contraddice uno dei corollari di $\left(*\right)$ poiché $\gamma+k\left\vert t\right\vert>0$.

Step 2.
Dimostriamo che $f$ è strettamente crescente nell'intervallo $\left(-\infty,\gamma\right]$.
Innanzitutto mostriamo che $f\left(-j+\gamma\right)<0\ \ \forall\ j>0$ (mostrando questo si avrà anche che per i due corollari di $\left(*\right)$ dovrà valere che $f\left(-j+\gamma\right)=-f\left(j-\gamma\right)\ \ \left(***\right)$).
Se $f\left(-j+\gamma\right)\ne f\left(j\right)$ allora siamo già a posto perché per $\left(**\right)$ si avrà che $$f\left(-j+\gamma\right)+f\left(j\right)=f\left(\gamma\right)\stackrel{f\left(j\right)>0}{\Rightarrow}f\left(-j+\gamma\right)<f\left(\gamma\right)<0$$quindi supponiamo invece di avere $f\left(-j+\gamma\right)=f\left(j\right)>0$. Allora per i due corollari di $\left(*\right)$ si ha che $f\left(j-\gamma\right)=f\left(j\right)>0$, ma allora per $\left(**\right)$ si avrà che $$f\left(j-\gamma\right)+f\left(\gamma\right)=f\left(j\right)$$il che è assurdo perché dovremmo avere che $f\left(\gamma\right)=0$.
Adesso scegliamo un $k$ generico tale che $k\le\gamma$ e dimostriamo che per ogni $j>0$ si ha che $f\left(-j+k\right)<f\left(k\right)$. Poiché abbiamo già fatto vedere che $f\left(-j+k\right)<0$, e poiché sappiamo per il secondo corollario di $\left(*\right)$ che $f\left(j\right)>0$, possiamo dire che $f\left(-j+k\right)\ne f\left(j\right)$, quindi per $\left(**\right)$ abbiamo che $f\left(-j+k\right)+f\left(j\right)=f\left(k\right)$, quindi avendo che $f\left(j\right)>0$, si ha che $f\left(-j+k\right)<f\left(k\right)$ che era quello che volevamo.
Ma allora per $\left(***\right)$ avremo anche che $f$ è strettamente crescente nell'intervallo $\left[\gamma,+\infty\right)$.

Step 3.
Dimostriamo ora la crescenza stretta nell'intervallo $\left(-\infty,+\infty\right)$. Supponiamo per assurdo che esistano $a<b:\ f\left(a\right)>f\left(b\right)$. Allora scegliamo $w$ opportunamente grande tale che $a+w>\gamma$ e tale che $f\left(a\right)\ne f\left(w\right)\ne f\left(b\right)$ (possiamo trovare tale $w$ grazie alla crescenza stretta da un certo punto in poi dimostrata nello Step 2.). Allora per $\left(**\right)$ si ha sia che $f\left(a+w\right)=f\left(a\right)+f\left(w\right)$, che $f\left(b+w\right)=f\left(b\right)+f\left(w\right)$, quindi $f\left(a+w\right)>f\left(b+w\right)$ e allo stesso tempo $a+w<b+w$, assurdo perché $a+w>\gamma$ e quindi questo contraddirebbe la crescenza stretta da $\gamma$ in poi dimostrata nello step 2.

Come corollario dello step 3. combinato con l'osservazione che $f\left(-x\right)=\pm f\left(x\right)$ si ha che sicuramente $f\left(-x\right)=-f\left(x\right)$.
Inoltre adesso la $\left(**\right)$ può essere riscritta equivalentemente nel modo seguente: $f\left(x\right)+f\left(y\right)=f\left(x+y\right)\ \ \forall\ x\ne y$. Ci manca solo da far vedere che $f\left(x\right)+f\left(x\right)=f\left(2x\right)\ \forall\ x\ne0$ per ridurci a una Cauchy (ok, è la funzione è da $\mathbb{R}$ in $\mathbb{R}$, ma abbiamo l'ipotesi aggiuntiva che è strettamente crescente)

Step 4.
Dimostriamo che $f\left(x\right)+f\left(x\right)=f\left(2x\right)\ \forall\ x\ne0$.
Notiamo che poiché $-x\ne 2x$ possiamo dire che $f\left(2x\right)+f\left(-x\right)=f\left(x\right)$, e poiché $f\left(-x\right)=-f\left(x\right)$ si ha proprio che $f\left(x\right)+f\left(x\right)=f\left(2x\right)$.

Da qui si conclude facilmente imponendo che $f$ sia una retta.
FloatingPoint
Messaggi: 22
Iscritto il: 18 lug 2016, 20:34

Re: Funzionale a caso

Messaggio da FloatingPoint »

bern-1-16-4-13 ha scritto: 04 giu 2017, 13:33 Ok, dopo numerosissime richieste, questa è una soluzione vera (o almeno dovrebbe) :lol:
Testo nascosto:
Sia $QUESTONONE'UNTROLL\left(x,y\right)$ l'uguaglianza del testo.
Step 0.
Ponendo $QUESTONONE'UNTROLL\left(0,y\right)$ si ottiene $f\left(0\right)=f\left(0\right)f\left(y\right)$, da cui $f\left(0\right)=0$ altrimenti $f\left(y\right)=1\ \ \forall y$ il che contraddirebbe la seconda ipotesi.
Ponendo $QUESTONONE'UNTROLL\left(x,x\right)$ si ottiene che $f\left(x^2\right)=f\left(x\right)^2\ \ \left(*\right)$, e da questa, notando che il LHS rimane invariato sostituendo $x$ con $-x$, si ottiene anche che $f\left(x\right)^2=f\left(-x\right)^2$, da cui $f\left(-x\right)=\pm f\left(x\right)$. Un altro corollario di $\left(*\right)$ è che $f\left(x\right)\ge 0\ \ \forall\ x\ge 0$.
Sostituendo nel testo la $\left(*\right)$ si ha che $$\left(f\left(x\right)-f\left(y\right)\right)\left(f\left(x\right)+f\left(y\right)\right)=f\left(x+y\right)\left(f\left(x\right)-f\left(y\right)\right)$$da cui $f\left(x\right)+f\left(y\right)=f\left(x+y\right)\ \ \forall\ x,y:\ f\left(x\right)\ne f\left(y\right)\ \ \left(**\right)$.

Step 1.
Dimostriamo che se $x\ne 0$ allora $f\left(x\right)\ne 0$ (quindi il secondo corollario di $\left(*\right)$ diventerebbe $\left(*\right)$ è che $f\left(x\right)>0\ \ \forall\ x>0$). Se per assurdo esistesse $t\ne 0:\ \ f\left(t\right)=0$ allora per il corollario della $\left(*\right)$ si ha che $f\left(\left\vert t\right\vert\right)=0$, ma allora scegliamo $\gamma$ tale che $f\left(\gamma\right)<0$ e un $k$ intero positivo tale che $k\left\vert t\right\vert>-\gamma$. Da una reiterazione della $\left(**\right)$ si ha che $f\left(\gamma\right)+kf\left(\left\vert t\right\vert\right)=f\left(\gamma+k\left\vert t\right\vert\right)$ da cui $f\left(\gamma+k\left\vert t\right\vert\right)=f\left(\gamma\right)<0$, assurdo perché contraddice uno dei corollari di $\left(*\right)$ poiché $\gamma+k\left\vert t\right\vert>0$.

Step 2.
Dimostriamo che $f$ è strettamente crescente nell'intervallo $\left(-\infty,\gamma\right]$.
Innanzitutto mostriamo che $f\left(-j+\gamma\right)<0\ \ \forall\ j>0$ (mostrando questo si avrà anche che per i due corollari di $\left(*\right)$ dovrà valere che $f\left(-j+\gamma\right)=-f\left(j-\gamma\right)\ \ \left(***\right)$).
Se $f\left(-j+\gamma\right)\ne f\left(j\right)$ allora siamo già a posto perché per $\left(**\right)$ si avrà che $$f\left(-j+\gamma\right)+f\left(j\right)=f\left(\gamma\right)\stackrel{f\left(j\right)>0}{\Rightarrow}f\left(-j+\gamma\right)<f\left(\gamma\right)<0$$quindi supponiamo invece di avere $f\left(-j+\gamma\right)=f\left(j\right)>0$. Allora per i due corollari di $\left(*\right)$ si ha che $f\left(j-\gamma\right)=f\left(j\right)>0$, ma allora per $\left(**\right)$ si avrà che $$f\left(j-\gamma\right)+f\left(\gamma\right)=f\left(j\right)$$il che è assurdo perché dovremmo avere che $f\left(\gamma\right)=0$.
Adesso scegliamo un $k$ generico tale che $k\le\gamma$ e dimostriamo che per ogni $j>0$ si ha che $f\left(-j+k\right)<f\left(k\right)$. Poiché abbiamo già fatto vedere che $f\left(-j+k\right)<0$, e poiché sappiamo per il secondo corollario di $\left(*\right)$ che $f\left(j\right)>0$, possiamo dire che $f\left(-j+k\right)\ne f\left(j\right)$, quindi per $\left(**\right)$ abbiamo che $f\left(-j+k\right)+f\left(j\right)=f\left(k\right)$, quindi avendo che $f\left(j\right)>0$, si ha che $f\left(-j+k\right)<f\left(k\right)$ che era quello che volevamo.
Ma allora per $\left(***\right)$ avremo anche che $f$ è strettamente crescente nell'intervallo $\left[\gamma,+\infty\right)$.

Step 3.
Dimostriamo ora la crescenza stretta nell'intervallo $\left(-\infty,+\infty\right)$. Supponiamo per assurdo che esistano $a<b:\ f\left(a\right)>f\left(b\right)$. Allora scegliamo $w$ opportunamente grande tale che $a+w>\gamma$ e tale che $f\left(a\right)\ne f\left(w\right)\ne f\left(b\right)$ (possiamo trovare tale $w$ grazie alla crescenza stretta da un certo punto in poi dimostrata nello Step 2.). Allora per $\left(**\right)$ si ha sia che $f\left(a+w\right)=f\left(a\right)+f\left(w\right)$, che $f\left(b+w\right)=f\left(b\right)+f\left(w\right)$, quindi $f\left(a+w\right)>f\left(b+w\right)$ e allo stesso tempo $a+w<b+w$, assurdo perché $a+w>\gamma$ e quindi questo contraddirebbe la crescenza stretta da $\gamma$ in poi dimostrata nello step 2.

Come corollario dello step 3. combinato con l'osservazione che $f\left(-x\right)=\pm f\left(x\right)$ si ha che sicuramente $f\left(-x\right)=-f\left(x\right)$.
Inoltre adesso la $\left(**\right)$ può essere riscritta equivalentemente nel modo seguente: $f\left(x\right)+f\left(y\right)=f\left(x+y\right)\ \ \forall\ x\ne y$. Ci manca solo da far vedere che $f\left(x\right)+f\left(x\right)=f\left(2x\right)\ \forall\ x\ne0$ per ridurci a una Cauchy (ok, è la funzione è da $\mathbb{R}$ in $\mathbb{R}$, ma abbiamo l'ipotesi aggiuntiva che è strettamente crescente)

Step 4.
Dimostriamo che $f\left(x\right)+f\left(x\right)=f\left(2x\right)\ \forall\ x\ne0$.
Notiamo che poiché $-x\ne 2x$ possiamo dire che $f\left(2x\right)+f\left(-x\right)=f\left(x\right)$, e poiché $f\left(-x\right)=-f\left(x\right)$ si ha proprio che $f\left(x\right)+f\left(x\right)=f\left(2x\right)$.

Da qui si conclude facilmente imponendo che $f$ sia una retta.
Tutto a posto, a parte un typo:
Testo nascosto:
nello Step 3 e alla fine dello Step 2, hai scritto [math] anziché [math].
Vediamo se riuscirai a risolvere il problema che posterò in Geometria...
Avatar utente
karlosson_sul_tetto
Messaggi: 1452
Iscritto il: 10 set 2009, 13:21
Località: Napoli

Re: Funzionale a caso

Messaggio da karlosson_sul_tetto »

Posto anche la mia soluzione, non perché sia sostanzialmente diversa ma perché l'avevo già scritta in gran parte e mi dispiaceva lasciarla marcire.
In compenso risolverò il problema nella sua versione generale, ovvero senza la seconda condizione.
Testo nascosto:
Chiamo (1) la formula del testo. Ponendo $x=y$ si ottiene:
$ f(x^2)-f(x)^2=f(x+x)(f(x)-f(x))=0 \; \; \rightarrow \; f(x^2)=f(x)^2 \;\;\; $ (2)

Con $x$ e $-x$ si ottiene $f(-x)^2=f((-x)^2)=f(x^2)=f(x)^2$ da cui $f(x)=\pm f(-x) \; \; \forall x\;$ (3)

Sostituendo prima $f(x^2)$ in (1) si ottiene:
$f(x+y)(f(x)-f(y))=f(x^2)-f(y)^2=f(x)^2-f(y)^2=(f(x)-f(y))(f(x)+f(y))\; \;$ (formula (1a) )
$(f(x)-f(y))(f(x)+f(y)-f(x+y))=0$
Da cui si ottiene che se $f(x)\neq f(y)$ allora $f(x+y)=f(x)+f(y)\; \;$ (4)

Sostitunedo invece $f(y)^2$ in (1) si ottiene:
$f(x+y)(f(x)-f(y))=f(x^2)-f(y)^2=f(x^2)-f(y^2)$; qui noto che il membro di destra non cambio se metto $\pm x$ o $\pm y$, dunque ho quattro combinazioni possibili: (chiamo due qualsiasi tra queste uguaglianze come (5) )
$f(x^2)-f(y^2)=f(x+y)(f(x)-f(y))=f(x-y)(f(x)-f(-y))=f(-x+y)(f(-x)-f(y))=f(-x-y)(f(-x)-f(-y))$


Con $x=0$ in (1) ottengo $f(0)^2=f(0)$, da cui $f(0)=0,1$. Analizzo il caso in cui $f(0)=1$; presuppongo $\exists\;\; x \mid f(x)\neq 1$, dunque sostituendo $(x,0)$ in (4) la condizione $f(x)\neq f(y)$ è verificata e si ha $f(x)=f(x+0)=f(x)+f(0)=f(x)+1$ che è un assurdo. Dunque $\forall x$ si ha che $f(x)=1$.
Sostituendo vedo che questa soluzione costante verifica (1) ma non l'ipotesi, quindi è soluzione solo del problema generale.

Quindi adesso sto nel caso in cui $f(0)=0$. Ricordo che la (3) ci dice $f(-x)=\pm f(x)$, dunque partiziono $\mathbb{R}$ in tre sottoinsiemi disgiunti $P,D,Z$ tali che:
  • $x \in Z \iff f(x)=f(-x)=0$
  • $x \in P \iff f(x)= f(-x) \neq 0$
  • $x \in D \iff f(x)= -f(-x) \neq 0$
So certamente che $Z$ non è vuoto, poiché $0\in Z$. Se $Z=\mathbb{R}$, ho trovato una soluzione $f(x)=0$ che soddisfa (1); dunque uno tra $P$ e $D$ deve avere almeno un elemento, vedo cosa succede.

Se esiste $a \in D$, allora ponendo $y=a-x$ in (5) con i segni $+,+=-,-$:
$f(x+a-x)(f(x)-f(a-x))=f(-x-a+x)(f(-x)-f(x-a))$
Siccome $f(a)=-f(-a)\neq 0$, posso semplificarlo ottenendo
$f(x)+f(-x)=f(a-x)+f(x-a)$
Presuppongo esista $b\in P$; allora $f(-b)=f(b)$ e nell'espressione di sopra metto $x=b$ e $x=-b$, ottenendo:
$f(b)+f(-b)=f(a-b)+f(-(a-b))$ e $f(-b)+f(b)=f(a+b)+f(-(a+b))$
Il membro di sinistra è $f(b)+f(-b)=2f(b)\neq 0$ poiché $b \not \in Z$. Dunque $a-b$ e $a+b$ non possono appartenere a $D$ o a $Z$, perché in tal caso si avrebbe $f(a-b)+f(-(a-b))=f(a+b)+f(-(a+b))=0$, assurdo. Dunque $a-b,a+b \in P$ e $f(a-b)=f(a+b)=f(b)$.

Ora se esiste $c \in P$, allora in modo simile a sopra ponendo $y=c-x$ in (5) si ha:
$f(x+c-x)(f(x)-f(c-x))=f(-x-c+x)(f(-x)-f(-c+x))$
Siccome $f(c)=f(-c)\neq 0$ si può semplificare:
$f(x)-f(-x)=f(c-x)-f(x-c)$
Presuppongo esista $d \in D$, allora pongo $x=d$ e $x=-d$ (nella seconda cambio i segni per chiarezza):
$f(d)-f(-d)=f(c-d)-f(d-c)$ e $f(d)-f(-d)=f(-d-c)-f(c+d)$
$f(d)-f(-d)=f(d)-(-f(d))=2f(d)\neq 0$, quindi $f(c-d)-f(d-c),f(-d-c)-f(c+d)\neq 0$ dunque $c-d,c+d$ non possono appartenere né a $Z$ né a $P$, dunque appartengono a $D$ e $f(c-d)=f(-c-d)=f(d)$.

Ora presupponiamo che $P,D$ sono entrambi non vuoti: $\exists \;\; m \in P \wedge \exists\; \; n \in D$. Allora ponendo $a=n, b=m$ nel primo blocco di sopra si arriva a dire che $a+b=m+n\in P$; ponendo $c=m, d=n$ nel seconso si arriva a dire che $c+d=m+n\in D$, e questo è chiaramente un assurdo.
Quindi uno dei due insiemi è necessariamente vuoto; ho quindi da analizzare due casi.


1) L'insieme $P$ è quello vuoto, ovvero $\mathbb{R}=Z\cup D$ e $f(x)=-f(-x)$ (questo vale per sia per $x\in D$ che per $x \in Z$).
Noto che per (2) si ha $f(x^2)=f(x)^2\geq 0$, dunque preso $y>0$ si ha $f(y)=f(\sqrt{y}^2)=f(\sqrt{2})^2\geq 0$; visto che $f(-y)=-f(y)\leq 0$ si ottiene:
  • $x\geq 0\; \rightarrow\; f(x) \geq 0$
  • $x\leq 0\; \rightarrow\; f(x) \leq 0$
Inoltre poiché $P$ contiene almeno un elemento $z$, conterrà anche $-z$; dunque ci sarà almeno un valore positivo e uno negativo per $f$.

Presuppongo che esista $k\neq 0$ tale che $f(k)=0$; da questo segue che $f(-k)=-f(k)=0$, quindi presuppongo senza perdita di generalità che $k>0$.
Allora prendo $x\in P, x <0$; poiché $f(x)\neq 0$ posso applicare (4), ottenendo $f(x+k)=f(x)+f(k)=f(x)\neq0$. Ripetendo questo procedimento con $x+k$ al posto di $x$ si arriva a $f(x+nk)=f(x)$ per qualsiasi $n$ intero $\geq 0$. Ma visto che per $k>0$, per $n$ grandi $nk+x>0$ mentre $x<0$, dunque $f(nk+x)>0$ e $f(x)<0$ per il ragionamento di sopra (escludo il caso di uguaglianza perché $f(x)\neq 0$. Ma questo è assurdo perché dovrei avere un'uguaglianza.

Quindi $f(x)=0 \iff x=0$. Presuppongo esistano due valori $x\neq y$ tali che $f(x)=f(y)\neq 0$. Considero $f(x-y)$: se è diverso da $f(y)$, allora posso applicare (4) ottenendo $f(y)+f(x-y)=f(x)$; siccome $f(x)=f(y)$ segue $f(x-y)=0$,
ma ciò è possibile solo se l'argomento è uguale a 0, ovvero $x=y$, assurdo. Se invece $f(x-y)=f(y)$, allora $f(y-x)=-f(y-x)=-f(y)\neq f(x)$, dunque per (4) $f(y-x)+f(x)=f(y)$ da cui $f(y-x)=0$ e quindi $x=y$. In entrambi i casi ho ottenuto
che non ci possono essere due valori che abbiano la stessa immagine, ovvero $f(x)$ è iniettiva.

Adesso si può dire una cosa più forte su (4), ovvero: $f(x+y)=f(x)+f(y)$ per ogni $x\neq y$. Da questo segue $f(2x)+f(-x)=f(x)$; ovvero sostituendo $f(-x)=-f(x)$ si ha $f(2x)=2f(x)$. Questo unito al risultato di prima ci da:
$ f(x+y)=f(x)+f(y) \; \; \forall x,y \in \mathbb{R} $

Questa è una Cauchy che ci dice che per $x \in \mathbb{Q}$ si ha $f(x)=\alpha x$; siccome per $x>0$, $f(x)>0$ si può estendere ai reali, ovvero $f(x)=\alpha x$ per ogni $x$ reale. Sostituendo in (2) si ottiene:
$\alpha x^2=f(x^2)=f(x)^2=\alpha^2 x^2 \; \; \forall x$, da cui $\alpha =0,1$. Il primo caso si esclude perché c'era almeno un elemento in $P$, dunque $\alpha=1 $ e $f(x)=x$, che verifica entrambe le ipotesi richieste.



2) L'insieme $D$ è quello vuoto, ovvero $\mathbb{R}=Z\cup P$ e $f(x)=f(-x)$ (questo vale per sia per $x\in D$ che per $x \in Z$).
Visto che per $x>0$ si ha $f(x)\geq 0$, si ottiene che $f(x)\geq 0 \; \; \forall x$. Questo contraddice la seconda ipotesi del testo, quindi il problema originale finisce qui; tuttavia continueremo imperterriti nella risoluzione.

Prendo $a,b$ tali che $f(a)>f(b)$. Esistono sicuramente perché posso scegliere $b$ in modo che $f(b)=0$ e poiche $P$ ha almeno un elemento, $\exists a$ tale che $f(a)>0$.
Se $f(b-a)\neq f(a)$, allora dalla (4) si ottiene $f(b)=f(b-a)+f(a)\geq f(a) >f(b)$ poiché $f(b-a) \geq 0$, assurdo. Dunque $f(b-a)=f(a)=f(a-b)$ perché la funzione è pari. Essendo $f(a-b)=f(a)>f(b)$ si ha per la (4): $f(a-b)+f(b)=f(a)$, cioé $f(b)=f(a)-f(a-b)=0$.
Dunque la funzione può assumere solo due valori, $0$ e $k>0$: se ci fosse anche $j$ nel codominio, per la dimostrazione di sopra $\min (k,j)=0$. Prendo $a$ tale che $f(a)=k$; per la (2) $f(a^2)=f(a)^2=k^2$; siccome $f(a^2)=0,k$ e non può essere $0$, si ottiene che $k=k^2$ da cui $k=1$.

Quindi $f(x)=1$ se $x \in D$ e $f(x)=0$ altrimenti. Noto che la condizione (1) del testo è completamente equivalente a (2) e (4) (una volta ottenuta (2), se $f(x)\neq f(y)$ allora necessariamente $f(x+y)=f(x)+f(y)$). Quindi per risolvere il problema mi basta caratterizzare tutti gli insiemi $D$ e $Z$ in cui posso partizionare $\mathbb{R}$ in modo che vengano rispettate le seguenti condizioni:
  • $x \in D \iff -x \in D\; \; \; \; \; $ $x \in Z \iff -x \in Z$
  • $x\in D \iff x^2 \in D\; \; \; \; \; $ $x\in Z \iff x^2 \in Z$ (derivante da (2) perché se $x\in D\rightarrow f(x^2)=f(x)^2=1$)
  • $\forall x \in D, y \in Z, x+y\in D$ (la (4) può essere applicata solo se $x,y$ appartengono ad insiemi diversi; la somma delle funzioni è $f(x)+f(y)=1+0=1$, dunque $f(x+y)=1$)
Presupponiamo che esistano $y_1,y_2 \in Z$ tali che $y_1+y_2\in D$; allora $(y_1+y_2)+ (-y_1)=y_2$ dove il primo elemento sta in $D$, il secondo in $Z$, dunque $y_2$ dovrebbe stare in $D$, assurdo. Quindi $\forall x,y \in Z \rightarrow x+y\in Z$, ovvero $Z$ è additivamente chiuso. Questa condizione è equivalente a quella di sopra: data la chiusura additiva se per certi $x\in D, y\in Z$ si ha $x+y\in Z$, allora $(x+y)+(-x)=y$ che dovrebbe stare in $D$, assurdo.

Quindi le richieste sono equivalenti a trovare un sottoinsieme $Z$ di $\mathbb{R}$ che sia additivamente chiuso, contenga lo 0, contenga l'opposto di ogni suo numero e che $x\in Z \iff x^2 \in Z$.

E questo lo possiamo costruire esplicitamente (ok non è vero sto barando, ma facciamo finta che lo sia): presa un'opportuna base di Hamel $(b_0,b_1,b_2\ldots)$, aggiungo a $Z$ lo $0$ e tutti gli elementi della forma $\sum\limits_{a,n\in \mathbb{Z}}a\cdot b_0^{2^n}$; i numeri che ottengo possono essere o già ottenuti in modo più facile oppure essere "irrazionali nuovi". In tal caso scelgo la base di Hamel in modo che contenga tutti gli irrazionali nuovi (o modifico quella precedente in modo da poterli inserire al posto di altra roba più inutile), quindi continuo a fare combinazioni del tipo $\sum\limits_{a,n,i\in\mathbb{Z}}a\cdot b_i^{2^n}$.

Questa caratterizzazione fa abbastanza schifo, ma so per certezza che esistono soluzioni non banali (ovvero $Z={0}$ o $Z=\mathbb{R}$): scegliendo $b_0=1$ allora posso ottenere tanti numeri algebrici grazie a $\sum\limits_{a,n,i\in\mathbb{Z}}a\cdot b_i^{2^n}$, ma non potrò mai ottenere numeri trascendentali.


In conclusioni, l'unica soluzione alla funzionale con entrambe le condizioni è $f(x)=x$, mentre solo alla prima condizione sono $f(x)=x, f(x)=1,f(x)=0$ e $\begin{cases} f(x)=0 \iff x \in Z \\ f(x)=1 \iff x \not\in Z\end{cases}$ con $Z$ descritto come l'obbrobrio qui sopra.
"Inequality happens"
---
"Chissa se la fanno anche da asporto"
FloatingPoint
Messaggi: 22
Iscritto il: 18 lug 2016, 20:34

Re: Funzionale a caso

Messaggio da FloatingPoint »

karlosson_sul_tetto ha scritto: 07 giu 2017, 23:36 Posto anche la mia soluzione, non perché sia sostanzialmente diversa ma perché l'avevo già scritta in gran parte e mi dispiaceva lasciarla marcire.
In compenso risolverò il problema nella sua versione generale, ovvero senza la seconda condizione.
Testo nascosto:
Chiamo (1) la formula del testo. Ponendo $x=y$ si ottiene:
$ f(x^2)-f(x)^2=f(x+x)(f(x)-f(x))=0 \; \; \rightarrow \; f(x^2)=f(x)^2 \;\;\; $ (2)

Con $x$ e $-x$ si ottiene $f(-x)^2=f((-x)^2)=f(x^2)=f(x)^2$ da cui $f(x)=\pm f(-x) \; \; \forall x\;$ (3)

Sostituendo prima $f(x^2)$ in (1) si ottiene:
$f(x+y)(f(x)-f(y))=f(x^2)-f(y)^2=f(x)^2-f(y)^2=(f(x)-f(y))(f(x)+f(y))\; \;$ (formula (1a) )
$(f(x)-f(y))(f(x)+f(y)-f(x+y))=0$
Da cui si ottiene che se $f(x)\neq f(y)$ allora $f(x+y)=f(x)+f(y)\; \;$ (4)

Sostitunedo invece $f(y)^2$ in (1) si ottiene:
$f(x+y)(f(x)-f(y))=f(x^2)-f(y)^2=f(x^2)-f(y^2)$; qui noto che il membro di destra non cambio se metto $\pm x$ o $\pm y$, dunque ho quattro combinazioni possibili: (chiamo due qualsiasi tra queste uguaglianze come (5) )
$f(x^2)-f(y^2)=f(x+y)(f(x)-f(y))=f(x-y)(f(x)-f(-y))=f(-x+y)(f(-x)-f(y))=f(-x-y)(f(-x)-f(-y))$


Con $x=0$ in (1) ottengo $f(0)^2=f(0)$, da cui $f(0)=0,1$. Analizzo il caso in cui $f(0)=1$; presuppongo $\exists\;\; x \mid f(x)\neq 1$, dunque sostituendo $(x,0)$ in (4) la condizione $f(x)\neq f(y)$ è verificata e si ha $f(x)=f(x+0)=f(x)+f(0)=f(x)+1$ che è un assurdo. Dunque $\forall x$ si ha che $f(x)=1$.
Sostituendo vedo che questa soluzione costante verifica (1) ma non l'ipotesi, quindi è soluzione solo del problema generale.

Quindi adesso sto nel caso in cui $f(0)=0$. Ricordo che la (3) ci dice $f(-x)=\pm f(x)$, dunque partiziono $\mathbb{R}$ in tre sottoinsiemi disgiunti $P,D,Z$ tali che:
  • $x \in Z \iff f(x)=f(-x)=0$
  • $x \in P \iff f(x)= f(-x) \neq 0$
  • $x \in D \iff f(x)= -f(-x) \neq 0$
So certamente che $Z$ non è vuoto, poiché $0\in Z$. Se $Z=\mathbb{R}$, ho trovato una soluzione $f(x)=0$ che soddisfa (1); dunque uno tra $P$ e $D$ deve avere almeno un elemento, vedo cosa succede.

Se esiste $a \in D$, allora ponendo $y=a-x$ in (5) con i segni $+,+=-,-$:
$f(x+a-x)(f(x)-f(a-x))=f(-x-a+x)(f(-x)-f(x-a))$
Siccome $f(a)=-f(-a)\neq 0$, posso semplificarlo ottenendo
$f(x)+f(-x)=f(a-x)+f(x-a)$
Presuppongo esista $b\in P$; allora $f(-b)=f(b)$ e nell'espressione di sopra metto $x=b$ e $x=-b$, ottenendo:
$f(b)+f(-b)=f(a-b)+f(-(a-b))$ e $f(-b)+f(b)=f(a+b)+f(-(a+b))$
Il membro di sinistra è $f(b)+f(-b)=2f(b)\neq 0$ poiché $b \not \in Z$. Dunque $a-b$ e $a+b$ non possono appartenere a $D$ o a $Z$, perché in tal caso si avrebbe $f(a-b)+f(-(a-b))=f(a+b)+f(-(a+b))=0$, assurdo. Dunque $a-b,a+b \in P$ e $f(a-b)=f(a+b)=f(b)$.

Ora se esiste $c \in P$, allora in modo simile a sopra ponendo $y=c-x$ in (5) si ha:
$f(x+c-x)(f(x)-f(c-x))=f(-x-c+x)(f(-x)-f(-c+x))$
Siccome $f(c)=f(-c)\neq 0$ si può semplificare:
$f(x)-f(-x)=f(c-x)-f(x-c)$
Presuppongo esista $d \in D$, allora pongo $x=d$ e $x=-d$ (nella seconda cambio i segni per chiarezza):
$f(d)-f(-d)=f(c-d)-f(d-c)$ e $f(d)-f(-d)=f(-d-c)-f(c+d)$
$f(d)-f(-d)=f(d)-(-f(d))=2f(d)\neq 0$, quindi $f(c-d)-f(d-c),f(-d-c)-f(c+d)\neq 0$ dunque $c-d,c+d$ non possono appartenere né a $Z$ né a $P$, dunque appartengono a $D$ e $f(c-d)=f(-c-d)=f(d)$.

Ora presupponiamo che $P,D$ sono entrambi non vuoti: $\exists \;\; m \in P \wedge \exists\; \; n \in D$. Allora ponendo $a=n, b=m$ nel primo blocco di sopra si arriva a dire che $a+b=m+n\in P$; ponendo $c=m, d=n$ nel seconso si arriva a dire che $c+d=m+n\in D$, e questo è chiaramente un assurdo.
Quindi uno dei due insiemi è necessariamente vuoto; ho quindi da analizzare due casi.


1) L'insieme $P$ è quello vuoto, ovvero $\mathbb{R}=Z\cup D$ e $f(x)=-f(-x)$ (questo vale per sia per $x\in D$ che per $x \in Z$).
Noto che per (2) si ha $f(x^2)=f(x)^2\geq 0$, dunque preso $y>0$ si ha $f(y)=f(\sqrt{y}^2)=f(\sqrt{2})^2\geq 0$; visto che $f(-y)=-f(y)\leq 0$ si ottiene:
  • $x\geq 0\; \rightarrow\; f(x) \geq 0$
  • $x\leq 0\; \rightarrow\; f(x) \leq 0$
Inoltre poiché $P$ contiene almeno un elemento $z$, conterrà anche $-z$; dunque ci sarà almeno un valore positivo e uno negativo per $f$.

Presuppongo che esista $k\neq 0$ tale che $f(k)=0$; da questo segue che $f(-k)=-f(k)=0$, quindi presuppongo senza perdita di generalità che $k>0$.
Allora prendo $x\in P, x <0$; poiché $f(x)\neq 0$ posso applicare (4), ottenendo $f(x+k)=f(x)+f(k)=f(x)\neq0$. Ripetendo questo procedimento con $x+k$ al posto di $x$ si arriva a $f(x+nk)=f(x)$ per qualsiasi $n$ intero $\geq 0$. Ma visto che per $k>0$, per $n$ grandi $nk+x>0$ mentre $x<0$, dunque $f(nk+x)>0$ e $f(x)<0$ per il ragionamento di sopra (escludo il caso di uguaglianza perché $f(x)\neq 0$. Ma questo è assurdo perché dovrei avere un'uguaglianza.

Quindi $f(x)=0 \iff x=0$. Presuppongo esistano due valori $x\neq y$ tali che $f(x)=f(y)\neq 0$. Considero $f(x-y)$: se è diverso da $f(y)$, allora posso applicare (4) ottenendo $f(y)+f(x-y)=f(x)$; siccome $f(x)=f(y)$ segue $f(x-y)=0$,
ma ciò è possibile solo se l'argomento è uguale a 0, ovvero $x=y$, assurdo. Se invece $f(x-y)=f(y)$, allora $f(y-x)=-f(y-x)=-f(y)\neq f(x)$, dunque per (4) $f(y-x)+f(x)=f(y)$ da cui $f(y-x)=0$ e quindi $x=y$. In entrambi i casi ho ottenuto
che non ci possono essere due valori che abbiano la stessa immagine, ovvero $f(x)$ è iniettiva.

Adesso si può dire una cosa più forte su (4), ovvero: $f(x+y)=f(x)+f(y)$ per ogni $x\neq y$. Da questo segue $f(2x)+f(-x)=f(x)$; ovvero sostituendo $f(-x)=-f(x)$ si ha $f(2x)=2f(x)$. Questo unito al risultato di prima ci da:
$ f(x+y)=f(x)+f(y) \; \; \forall x,y \in \mathbb{R} $

Questa è una Cauchy che ci dice che per $x \in \mathbb{Q}$ si ha $f(x)=\alpha x$; siccome per $x>0$, $f(x)>0$ si può estendere ai reali, ovvero $f(x)=\alpha x$ per ogni $x$ reale. Sostituendo in (2) si ottiene:
$\alpha x^2=f(x^2)=f(x)^2=\alpha^2 x^2 \; \; \forall x$, da cui $\alpha =0,1$. Il primo caso si esclude perché c'era almeno un elemento in $P$, dunque $\alpha=1 $ e $f(x)=x$, che verifica entrambe le ipotesi richieste.



2) L'insieme $D$ è quello vuoto, ovvero $\mathbb{R}=Z\cup P$ e $f(x)=f(-x)$ (questo vale per sia per $x\in D$ che per $x \in Z$).
Visto che per $x>0$ si ha $f(x)\geq 0$, si ottiene che $f(x)\geq 0 \; \; \forall x$. Questo contraddice la seconda ipotesi del testo, quindi il problema originale finisce qui; tuttavia continueremo imperterriti nella risoluzione.

Prendo $a,b$ tali che $f(a)>f(b)$. Esistono sicuramente perché posso scegliere $b$ in modo che $f(b)=0$ e poiche $P$ ha almeno un elemento, $\exists a$ tale che $f(a)>0$.
Se $f(b-a)\neq f(a)$, allora dalla (4) si ottiene $f(b)=f(b-a)+f(a)\geq f(a) >f(b)$ poiché $f(b-a) \geq 0$, assurdo. Dunque $f(b-a)=f(a)=f(a-b)$ perché la funzione è pari. Essendo $f(a-b)=f(a)>f(b)$ si ha per la (4): $f(a-b)+f(b)=f(a)$, cioé $f(b)=f(a)-f(a-b)=0$.
Dunque la funzione può assumere solo due valori, $0$ e $k>0$: se ci fosse anche $j$ nel codominio, per la dimostrazione di sopra $\min (k,j)=0$. Prendo $a$ tale che $f(a)=k$; per la (2) $f(a^2)=f(a)^2=k^2$; siccome $f(a^2)=0,k$ e non può essere $0$, si ottiene che $k=k^2$ da cui $k=1$.

Quindi $f(x)=1$ se $x \in D$ e $f(x)=0$ altrimenti. Noto che la condizione (1) del testo è completamente equivalente a (2) e (4) (una volta ottenuta (2), se $f(x)\neq f(y)$ allora necessariamente $f(x+y)=f(x)+f(y)$). Quindi per risolvere il problema mi basta caratterizzare tutti gli insiemi $D$ e $Z$ in cui posso partizionare $\mathbb{R}$ in modo che vengano rispettate le seguenti condizioni:
  • $x \in D \iff -x \in D\; \; \; \; \; $ $x \in Z \iff -x \in Z$
  • $x\in D \iff x^2 \in D\; \; \; \; \; $ $x\in Z \iff x^2 \in Z$ (derivante da (2) perché se $x\in D\rightarrow f(x^2)=f(x)^2=1$)
  • $\forall x \in D, y \in Z, x+y\in D$ (la (4) può essere applicata solo se $x,y$ appartengono ad insiemi diversi; la somma delle funzioni è $f(x)+f(y)=1+0=1$, dunque $f(x+y)=1$)
Presupponiamo che esistano $y_1,y_2 \in Z$ tali che $y_1+y_2\in D$; allora $(y_1+y_2)+ (-y_1)=y_2$ dove il primo elemento sta in $D$, il secondo in $Z$, dunque $y_2$ dovrebbe stare in $D$, assurdo. Quindi $\forall x,y \in Z \rightarrow x+y\in Z$, ovvero $Z$ è additivamente chiuso. Questa condizione è equivalente a quella di sopra: data la chiusura additiva se per certi $x\in D, y\in Z$ si ha $x+y\in Z$, allora $(x+y)+(-x)=y$ che dovrebbe stare in $D$, assurdo.

Quindi le richieste sono equivalenti a trovare un sottoinsieme $Z$ di $\mathbb{R}$ che sia additivamente chiuso, contenga lo 0, contenga l'opposto di ogni suo numero e che $x\in Z \iff x^2 \in Z$.

E questo lo possiamo costruire esplicitamente (ok non è vero sto barando, ma facciamo finta che lo sia): presa un'opportuna base di Hamel $(b_0,b_1,b_2\ldots)$, aggiungo a $Z$ lo $0$ e tutti gli elementi della forma $\sum\limits_{a,n\in \mathbb{Z}}a\cdot b_0^{2^n}$; i numeri che ottengo possono essere o già ottenuti in modo più facile oppure essere "irrazionali nuovi". In tal caso scelgo la base di Hamel in modo che contenga tutti gli irrazionali nuovi (o modifico quella precedente in modo da poterli inserire al posto di altra roba più inutile), quindi continuo a fare combinazioni del tipo $\sum\limits_{a,n,i\in\mathbb{Z}}a\cdot b_i^{2^n}$.

Questa caratterizzazione fa abbastanza schifo, ma so per certezza che esistono soluzioni non banali (ovvero $Z={0}$ o $Z=\mathbb{R}$): scegliendo $b_0=1$ allora posso ottenere tanti numeri algebrici grazie a $\sum\limits_{a,n,i\in\mathbb{Z}}a\cdot b_i^{2^n}$, ma non potrò mai ottenere numeri trascendentali.


In conclusioni, l'unica soluzione alla funzionale con entrambe le condizioni è $f(x)=x$, mentre solo alla prima condizione sono $f(x)=x, f(x)=1,f(x)=0$ e $\begin{cases} f(x)=0 \iff x \in Z \\ f(x)=1 \iff x \not\in Z\end{cases}$ con $Z$ descritto come l'obbrobrio qui sopra.
Bella soluzione, la (5) è una strada che avevo provato anche io, per poi abbandonarla. Non sono convintissimo della soluzione patologica che si ottiene senza la seconda ipotesi, ma sembra giusta.
Rispondi